Difference between revisions of "Mock AIME 4 2006-2007 Problems/Problem 14"

 
 
(One intermediate revision by one other user not shown)
Line 3: Line 3:
 
==Solution==
 
==Solution==
  
{{solution}}
+
We will assume that there is at least one solution, otherwise the answer would be undefined.
  
 +
Using the binomial theorem it is obvious that <math>(577-k)^4 \equiv k^4 \pmod {577}</math>. Thus the solutions come in pairs <math>\{k,577-k\}</math>, and hence their average is <math>\dfrac{577}2 = 288.5</math>, and the answer is <math>\boxed{288}</math>.
 +
 +
(In this case, there are four solutions: <math>276</math>, <math>277</math>, <math>300</math>, and <math>301</math>.)
 +
 +
----
 +
 +
*[[Mock AIME 4 2006-2007 Problems/Problem 15| Next Problem]]
 +
*[[Mock AIME 4 2006-2007 Problems/Problem 13| Previous Problem]]
 
*[[Mock AIME 4 2006-2007 Problems]]
 
*[[Mock AIME 4 2006-2007 Problems]]

Latest revision as of 01:52, 31 January 2009

Problem

Let $x$ be the arithmetic mean of all positive integers $k<577$ such that

$k^4\equiv 144\pmod {577}$.

Find the greatest integer less than or equal to $x$.

Solution

We will assume that there is at least one solution, otherwise the answer would be undefined.

Using the binomial theorem it is obvious that $(577-k)^4 \equiv k^4 \pmod {577}$. Thus the solutions come in pairs $\{k,577-k\}$, and hence their average is $\dfrac{577}2 = 288.5$, and the answer is $\boxed{288}$.

(In this case, there are four solutions: $276$, $277$, $300$, and $301$.)